Answer:
The answer to this question is 19.
Step-by-step explanation:
Given that :
f(x)=13.
f'(x)=3. 1 ≤ x ≤ 3.
Integrate
∫f'(x) dx=∫3 dx
f(x)=3x+c 1 ≤ x ≤ 3.
f(1)= 3+c
c=13-3 =10.
f(x)=3x+10 1 ≤ x ≤ 3.
now ,
f(3)=3(3)+10=19.
So f(3) is at least 19.
The LCM is the greater.
The GCF is a factor which divides in to each number so can never be equal to the greater number, whereas the LCM is the greater of the 2 numbers or a multiple of them
Answer:
144
Step-by-step explanation:
72% of 200 is 144
Equation: 0.72*200 = 144
Answer:
(B) 
General Formulas and Concepts:
<u>Calculus</u>
Limits
Derivatives
- The definition of a derivative is the slope of the tangent line.
Derivative Notation
Instantaneous Rates
- Tangent Line:

Step-by-step explanation:
Since we are trying to find a <em>rate</em> at which W(t) changes, we must find the <em>derivative</em> at <em>t</em> = 3.
We are given 2 close answer choices that would have the same <em>numerical</em> answer but different <em>meanings</em>:
- (A)

- (B)

If we look at answer choice (A), we see that our units would simply just be volume. It would not have the units of a rate of change. Yes, it may be the closest numerically correct answer, but it does not tell us the <em>rate</em> at which the volume would be changing and it is not a derivative.
If we look at answer choice (B), we see that our units would be cm³/s, and that is most certainly a rate of change. Answer choice (B) is also a <em>derivative</em> at <em>t</em> = 3, and a derivative tells us what <em>rate</em> something is changing.
∴ Answer choice (B) will give us the best estimate for the value of the instantaneous rate of change of W(t) when <em>t</em> = 3.
Topic: AP Calculus AB/BC (Calculus I/I + II)
Unit: Differentiation
Book: College Calculus 10e
Let Kaya's savings be 30x and Edgardo's savings be 35x
If they both started saving at the same time:
f(x)=30x
f(x)=35x
Now, sub in values for x in to the function starting with 0. Subtract y2-y1 and x2-x1 for both functions.
For slope: m=y2-y1/x2-x1
so your result will be m=30/1=30 for f(x) = 30x
and m=35/1=35 for f(x) = 35x
so the slopes are m=30 and m=35 respectively!